Difference between revisions of "2000 AMC 8 Problems/Problem 25"

(Solution 2)
(Solution 3)
 
(8 intermediate revisions by 5 users not shown)
Line 34: Line 34:
 
The above answer is fast, but satisfying, and assumes that the area of <math>\triangle AMN</math> is independent of the dimensions of the rectangle. All in all, it's a very good answer though. However this is an alternative if you don't get the above answer. Label <math>AB = CD = l</math> and <math>BC = DA = h</math>
 
The above answer is fast, but satisfying, and assumes that the area of <math>\triangle AMN</math> is independent of the dimensions of the rectangle. All in all, it's a very good answer though. However this is an alternative if you don't get the above answer. Label <math>AB = CD = l</math> and <math>BC = DA = h</math>
  
Labelling bob as the right and lower midpoints respectively, and redoing all the work above, we get:
+
Labelling <math>m</math> and <math>n</math> as the right and lower midpoints respectively, and redoing all the work above, we get:
  
<math>yeet = \frac{1}{2}\cdot h\cdot \frac{l}{2} = \frac{lh}{4}</math>
+
<math>[\triangle ABN] = \frac{1}{2}\cdot h\cdot \frac{l}{2} = \frac{lh}{4}</math>
  
 
<math>[\triangle MNC] = \frac{1}{2}\cdot \frac{l}{2}\cdot \frac{w}{2} = \frac{lh}{8}</math>
 
<math>[\triangle MNC] = \frac{1}{2}\cdot \frac{l}{2}\cdot \frac{w}{2} = \frac{lh}{8}</math>
Line 47: Line 47:
  
 
<math>[\triangle AMN] = \frac{3}{8}lh = \frac{3}{8}\cdot 72 = 27</math>, and the answer is <math>\boxed{B}</math>
 
<math>[\triangle AMN] = \frac{3}{8}lh = \frac{3}{8}\cdot 72 = 27</math>, and the answer is <math>\boxed{B}</math>
 +
 +
==Solution 3==
 +
Let's assume, for simplicity, that the sides of the rectangle are <math>9</math> and <math>8.</math> The area of the 3 triangles would then be <math>8\cdot\frac{9}{2}\cdot\frac{1}{2} = 18,</math>
 +
<math>4\cdot\frac{9}{2}\cdot\frac{1}{2} = 9,</math>
 +
<math>4\cdot 9\cdot\frac{1}{2} = 18.</math>
 +
Adding these up, we get <math>45</math>, and subtracting that from <math>72</math>, we get <math>27</math>, so the answer is <math>\boxed{B}</math>
 +
 +
~ilee0820
 +
 +
== Video Solution ==
 +
https://youtu.be/yoIO9q_GTig. Soo, DRMS, NM
 +
 +
https://www.youtube.com/watch?v=XxQwfirFn4M    ~David
  
 
==See Also==
 
==See Also==

Latest revision as of 13:59, 22 October 2023

Problem

The area of rectangle $ABCD$ is $72$ units squared. If point $A$ and the midpoints of $\overline{BC}$ and $\overline{CD}$ are joined to form a triangle, the area of that triangle is

[asy] pair A,B,C,D; A = (0,8); B = (9,8); C = (9,0); D = (0,0); draw(A--B--C--D--A--(9,4)--(4.5,0)--cycle); label("$A$",A,NW); label("$B$",B,NE); label("$C$",C,SE); label("$D$",D,SW);[/asy]

$\text{(A)}\ 21\qquad\text{(B)}\ 27\qquad\text{(C)}\ 30\qquad\text{(D)}\ 36\qquad\text{(E)}\ 40$

Solution 1

To quickly solve this multiple choice problem, make the (not necessarily valid, but very convenient) assumption that $ABCD$ can have any dimension. Give the rectangle dimensions of $AB = CD = 12$ and $BC = AD= 6$, which is the easiest way to avoid fractions. Labelling the right midpoint as $M$, and the bottom midpoint as $N$, we know that $DN = NC = 6$, and $BM = MC = 3$.

$[\triangle ADN] = \frac{1}{2}\cdot 6\cdot 6 = 18$

$[\triangle MNC] = \frac{1}{2}\cdot 3\cdot 6 = 9$

$[\triangle ABM] = \frac{1}{2}\cdot 12\cdot 3 = 18$

$[\triangle AMN] = [\square ABCD] - [\triangle ADN] - [\triangle MNC] - [\triangle ABM]$

$[\triangle AMN] = 72 - 18 - 9 - 18$

$[\triangle AMN] = 27$, and the answer is $\boxed{B}$

Solution 2

The above answer is fast, but satisfying, and assumes that the area of $\triangle AMN$ is independent of the dimensions of the rectangle. All in all, it's a very good answer though. However this is an alternative if you don't get the above answer. Label $AB = CD = l$ and $BC = DA = h$

Labelling $m$ and $n$ as the right and lower midpoints respectively, and redoing all the work above, we get:

$[\triangle ABN] = \frac{1}{2}\cdot h\cdot \frac{l}{2} = \frac{lh}{4}$

$[\triangle MNC] = \frac{1}{2}\cdot \frac{l}{2}\cdot \frac{w}{2} = \frac{lh}{8}$

$[\triangle ABM] = \frac{1}{2}\cdot l\cdot \frac{h}{2} = \frac{lh}{4}$

$[\triangle AMN] = [\square ABCD] - [\triangle ADN] - [\triangle MNC] - [\triangle ABM]$

$[\triangle AMN] = lh - \frac{lh}{4} - \frac{lh}{8} - \frac{lh}{4}$

$[\triangle AMN] = \frac{3}{8}lh = \frac{3}{8}\cdot 72 = 27$, and the answer is $\boxed{B}$

Solution 3

Let's assume, for simplicity, that the sides of the rectangle are $9$ and $8.$ The area of the 3 triangles would then be $8\cdot\frac{9}{2}\cdot\frac{1}{2} = 18,$ $4\cdot\frac{9}{2}\cdot\frac{1}{2} = 9,$ $4\cdot 9\cdot\frac{1}{2} = 18.$ Adding these up, we get $45$, and subtracting that from $72$, we get $27$, so the answer is $\boxed{B}$

~ilee0820

Video Solution

https://youtu.be/yoIO9q_GTig. Soo, DRMS, NM

https://www.youtube.com/watch?v=XxQwfirFn4M ~David

See Also

2000 AMC 8 (ProblemsAnswer KeyResources)
Preceded by
Problem 24
Followed by
Last
Question
1 2 3 4 5 6 7 8 9 10 11 12 13 14 15 16 17 18 19 20 21 22 23 24 25
All AJHSME/AMC 8 Problems and Solutions

The problems on this page are copyrighted by the Mathematical Association of America's American Mathematics Competitions. AMC logo.png